CVSR Quiz and BOFs Flashcards

1
Q
A

The correct answer is:

  1. The heart is enveloped by this fluid-filled structure → Pericardium,
  2. Contracts during ventricular systole to prevent regurgitation and leakage of blood back into the atria → Papillary muscle,
  3. Open during ventricular relaxation and filling → Mitral valve
How well did you know this?
1
Not at all
2
3
4
5
Perfectly
2
Q
A

The correct answer is:

  1. Slows conduction, ensuring atrial contraction is complete before ventricular contraction commences. → Atrioventricular node,
  2. Ensures rapid spread of electrical activity to the ventricular apex and into the cardiac muscle. → His – Purkinje system,
  3. By virtue of its intrinsic rate of depolarization, provides secondary back up of heart rate (junctional escape) if the primary pacemaker region is injured and inactivated. → Atrioventricular node
How well did you know this?
1
Not at all
2
3
4
5
Perfectly
3
Q
A

The correct answer is:

  1. The time between two consecutive instances of this component of the surface ECG is used to determine the heart rate → R wave,
  2. This component of the ECG represents the depolarisation of the atria. → P wave,
  3. This component of the ECG represents the plateau of the ventricular action potential. → S-T segment
How well did you know this?
1
Not at all
2
3
4
5
Perfectly
4
Q
A

The correct answer is:

  1. → Normal Sinus Rhythm,
  2. → Ventricular tachycardia,
  3. → Bundle branch block
How well did you know this?
1
Not at all
2
3
4
5
Perfectly
5
Q
A

The correct answers are: Foramen ovale, Ductus arteriosus, Ductus venosus, Umbilical vein, Foramen ovale, Atrioventricular canal

How well did you know this?
1
Not at all
2
3
4
5
Perfectly
6
Q
A

The correct answer is: Foramen ovale

How well did you know this?
1
Not at all
2
3
4
5
Perfectly
7
Q
A

The correct answer is: Pulmonary trunk, Aorta

How well did you know this?
1
Not at all
2
3
4
5
Perfectly
8
Q
A

The correct answer is: Low ductal response to O2

How well did you know this?
1
Not at all
2
3
4
5
Perfectly
9
Q
A

The correct answer is: Close during ventricular contraction to prevent the backflow of blood into the atrium

How well did you know this?
1
Not at all
2
3
4
5
Perfectly
10
Q
A

The correct answer is: Supraventricular crest

How well did you know this?
1
Not at all
2
3
4
5
Perfectly
11
Q
A

The correct answer is: Sinus venosus

How well did you know this?
1
Not at all
2
3
4
5
Perfectly
12
Q
A

The correct answer is: increased in females and decreased in males.

How well did you know this?
1
Not at all
2
3
4
5
Perfectly
13
Q
A

The correct answer is:

  1. A 73 year old man with a prior history of ischaemic heart disease is found dead in his flat. At autopsy there is evidence of scarring in the anterior 2/3 of the interventricular septum. → Left anterior descending artery,
  2. A 65 year old woman collapses in the mall and is unable to be resuscitated. At autopsy there is a cardiac tamponade with rupture of the lateral aspect of the left ventricular wall. → Left marginal artery,
  3. A 55 year old smoker collapses at the golf course. He had a history of episodes of angina. He could not be rescuscitated. At autopsy there were multiple areas of infarction in the right atrium and right ventricle. → Right coronary artery
How well did you know this?
1
Not at all
2
3
4
5
Perfectly
14
Q
A
  1. Endothelial damage provides the stimulus for the lipid deposition that is atherosclerosis.
  2. Cytokines and chemokines are the cellular attractants produced during inflammatory responses
  3. C-reactive protein is an inflammatory marker rises as an early sign of inflammation.

The correct answer is: 1. Atherosclerosis is initiated by: → Endothelial damage, 2. Cells are attracted to the atherosclerotic site due to: → Cytokines & chemokines, 3. Measurement of this parameter can be used in the assessment of risk of atherosclerosis: → C-reactive protein

How well did you know this?
1
Not at all
2
3
4
5
Perfectly
15
Q
A

Endothelial dysfunction or damage is the key initiating event in atherosclerosis and be triggered by: mechanical shear stress (e.g. hypertension), biochemical abnormalities (e.g. hyperlipidemia, diabetes), immunological factors, inflammation, toxic substances (e.g. cigarette smoke), free radicals or genetic factors (e.g. familial hypertension)

The correct answer is: Endothelial dysfunction/damage

How well did you know this?
1
Not at all
2
3
4
5
Perfectly
16
Q
A

Thrombi can form on the surface of a plaque or inside it. This usually results from plaque rupture or superficial injury, exposing the inside of the plaque which is highly thrombogenic (due to the presence of tissue factor). Thrombi can further obstruct the vessel by increasing the volume of the plaque & changing its shape (if internal) or leading to thromboembolism (if external).

The correct answer is: Thrombus formation

How well did you know this?
1
Not at all
2
3
4
5
Perfectly
17
Q
A

The correct answer is:

  1. Blood flow to individual organs is primarily determined by: → Vascular resistance, 2. Korotkoff sounds heard during auscultatory determination of blood pressure represent: → Blood turbulence,
  2. At the end of systole, retrograde blood flow to the left ventricle is prevented by → Aortic valve closure
How well did you know this?
1
Not at all
2
3
4
5
Perfectly
18
Q
A

The conus arteriosus is the smooth walled section in the right ventricle leading to the semilunar valves of the pulmonary trunk. It assists in directing blood flow through the pulmonary semilunar valves.

The correct answer is: is the vestibule leading to the pulmonary semilunar valves in the right ventricle.

How well did you know this?
1
Not at all
2
3
4
5
Perfectly
19
Q
A

The apex of the heart is at the level of the 5th intercostal space on the midclavicular line.

The correct answer is: 5th intercostal space on the midclavicular line

How well did you know this?
1
Not at all
2
3
4
5
Perfectly
20
Q
A

The papillary muscles contract during ventricular contraction to maintain tension in the chordae tendinae and prevent the cusps of the atrioventricular valves from protruding into the atria with the force of ventricular contraction. This prevents backflow of blood into the atria. Infarction would lead to failure of the valve leaflets to be properly held in place and hence sounds (murmur) associated with valve insufficiency (regurgitation of blood from ventricle to atrium) during systole.

The correct answer is: contract to prevent the atrioventricular valves from protruding into the atria.

How well did you know this?
1
Not at all
2
3
4
5
Perfectly
21
Q
A
  1. Air movement into and out of the lungs is dependent upon the pressure difference between atmosphere and lungs (alveoli). Neither the relative partial pressures of O2 (Po2) and CO2 (Pco2) nor cardiac output and muscle blood flow play any part in this. Air movement will not be influenced by how much oxygen is in the atmosphere or in the lungs or blood.
  2. For CO2 to leave the body, it must cross from the blood to the lungs. As muscle metabolism increases, the Pco2 in the blood will increase. Thus the pulmonary artery Pco2 will rise well above the alveolar Pco2, driving CO2 out.
  3. Oxygen will leave the blood and diffuse into the muscle if the Po2 in capillary blood is greater than the muscle Po2. A higher cardiac output and higher blood pressure will deliver more blood to the muscle; however it is the concentration of oxygen within the blood when it arrives at the muscle capillaries (Po2) the oxygen will not leave the capillary if the capillary Po2 is not greater than the muscle Po2. Capillary oncotic pressure will affect water movement.

The correct answer is: 1. What is the driving force for air entering the lungs? → Atmospheric pressure, 2. What is the driving force for removing CO2 from the body? → Pulmonary artery Pco2, 3. What is the driving force for O2 to enter the active muscle? → Capillary Po2

How well did you know this?
1
Not at all
2
3
4
5
Perfectly
22
Q
A

Atherosclerosis and Marfan’s syndrome are the most common causes of abdominal aneurysm. His stature likely excludes Marfan’s syndrome and his smoking history and presence of coronary artery disease (angina) suggest atherosclerosis. Cystic medial degeneration describes the lesion associated with Marfan’s syndrome. Extra-vascuilar haematoma is not a “true aneurysm”. There is no indication of infection andinfection only accounts for <1% of abdominal aneurysms

The correct answer is: Atherosclerosis

How well did you know this?
1
Not at all
2
3
4
5
Perfectly
23
Q
A
  1. A split R-wave is indicative of a slowed conduction in one branch of the His-Purkinje system such that the muscle cells of one ventricle are activated and depolarise earlier the other, rather than the usual simultaneous activation
  2. A prolonged P-R interval is indicative of slowed conduction through the AV node. If too slow, the AV node may take over pace-making and the QRS complex occur regularly but will no longer be synchronised with each P wave
  3. An absent P wave but a regular QRS complex indicates failure of the S-A node to depolarise. The A_V node takes over as secondary pacemaker but at a slower than normal rate

The correct answer is: 1. The ECG record shows a persistent double peak or “split” in the R-wave → His – Purkinje system, 2. The QRS complex is separated from the P wave by a larger than normal P-R interval → Atrioventricular node, 3. A regular QRS complex occurs but the P wave is absent and R-R interval is prolonged and the heart rate slowed at rest → Sinoatrial node

How well did you know this?
1
Not at all
2
3
4
5
Perfectly
24
Q
A

The correct answer is: Foramen ovale

How well did you know this?
1
Not at all
2
3
4
5
Perfectly
25
Q
A

The correct answer is: Glycolysis is increased and calcium ions are overloaded in the contractile cell

How well did you know this?
1
Not at all
2
3
4
5
Perfectly
26
Q
A

β1-adrenoceptors mediate sympathetic NS stimulation of myocardial contractility . Increasing the diastolic length of the myocytes will increase force of contraction but contractility is defined as increased force of contraction at a given length. Contractility represents greater Ca2+ release and an increased use of energy; increased length takes advantage of more crossbridges being made and does not require more Ca2+ or more energy. Parasymathetic NS does not innervate the myocytes and decreased PS acticity will increase heart rate but not contractility

The correct answer is: Stimulation β1-adrenoceptors on myocytes and increased calcium permeability

How well did you know this?
1
Not at all
2
3
4
5
Perfectly
27
Q
A

The tunica media consists of the many smooth muscle cells required to contract and dilate with collagen and elastin that hold the muscle cells together and against high pressures.

Arteries and arterioles have substantial elastic laminar (internal and external) to resist overstretching and provide rebound in response to pulse pressure; they have extensive vasa vasorum to deliver blood and nutrients into the thicker, more muscular tunica media; Endothelial cells play a significant role in resistance vessels in their response to sheer stress and vasoactive circulating substances, however it is not a thicker layer.

Refer Lecture: Blood vessel and endothelial structure and function.

The correct answer is: more substantial tunica media layer of cells.

How well did you know this?
1
Not at all
2
3
4
5
Perfectly
28
Q
A

Valve vegetative growth will often give rise to emboli and the consequences depend on the location of the valves. Mitral and aortic valves are on the arterial side, increasing risk of stroke (via carotid arteries). Emboli having this point of origin can never pass into the venous system or the pulmonary system. Emboli will cause localised increases in resistance but with little effect on total peripheral resistance. Valve dysfunction may ultimately lead to heart failure and low BP rather than hypertension.

Refer GOAL: Cardiovascular histopathology

The correct answer is: Arterial embolism causing ischaemic stroke

How well did you know this?
1
Not at all
2
3
4
5
Perfectly
29
Q
A

The correct answer is:

  1. Which of the ECG disorders is commonly observed in left ventricular hypertrophy? → Enlarged QRS complex,
  2. Which of the ECG disorders is commonly observed in bundle branch block? → Split R wave,
  3. Which of the ECG disorders is commonly observed in the early stages of a myocardial infarction? → ST segment depression
How well did you know this?
1
Not at all
2
3
4
5
Perfectly
30
Q
A

The correct answer is:
1. Name the vessel that supplies blood to the posterior wall of the left ventricle? → Circumflex artery,

  1. Name the vessel that lies in the anterior interventricular groove, starts near the apex and joins the coronary sinus at its left end? → Great cardiac vein,
  2. The coronary arteries originate from which vessel? → Ascending aorta,
  3. Deoxygenated blood returns from the upper body to the right atrium of the heart via which vessel? → Superior vena cava,
  4. Blockage leading to ischaemia most commonly occurs in which vessel? → Left anterior descending (LAD) artery,
  5. The posterior interventricular artery supplies oxygenated blood to the interventricular septum. What vessel returns deoxygenated blood from the same region? → Middle cardiac vein,
  6. Name the vessel that returns oxygenated blood from the lungs to the heart? → Pulmonary vein,
  7. Cardiac surgeons pass a finger through the transverse pericardial sinus to allow the application of a surgical clamp or ligature in order to stop or divert the circulation of blood in ascending aorta and which of the above vessels? → Pulmonary trunk
How well did you know this?
1
Not at all
2
3
4
5
Perfectly
31
Q
A

The correct answer is:
1. Further investigation showing elevated levels of which serum marker suggests that he has suffered ischaemic myocardial damage ? → Troponins,

  1. Echocardiogaphic investigation shows a region of ventricular free wall that does not fully relax, which might suggest reduced activity of which circulating or intracellular component in that region? → Sarcoplasmic reticulum ATPase,
  2. The extent of damage is quite severe and with time his condition progresses into heart failure. Drug treatment to reduce activity of what intracellular or circulating component will indirectly increase force of contraction through facilitation of intracellular Ca++ accumulation? → Na+/K+ ATPase
How well did you know this?
1
Not at all
2
3
4
5
Perfectly
32
Q
A

Coagulative necrosis only becomes visible 8-12 hours after the infarct

The correct answer is: Normal appearances

How well did you know this?
1
Not at all
2
3
4
5
Perfectly
33
Q
A

Dressler syndrome is a type of pericarditis, inflammation of the sac surrounding the heart (pericardium). The inflammation is believed to be an immune system response following damage to heart tissue or the pericardium, such as a myocardial infarction, surgery or traumatic injury. The fever is the give-away sign.

The correct answer is: Pericarditis - Dressler syndrome

How well did you know this?
1
Not at all
2
3
4
5
Perfectly
34
Q
A

Coronary blood flow increases in response to increased metabolic demand, via coronary artery vasodilatation. Oxygen extraction is already very high in coronary circulation. Breathing rate will increase, but this just serves to maintain oxygen saturation in the face of the the higher pulmonary blood flow. Cardiac output will increase to deliver more blood to skeletal muscle, but coronary flow will not increase unless the coronary arterioles dilate as well.
The correct answer is: Increased coronary blood flow

How well did you know this?
1
Not at all
2
3
4
5
Perfectly
35
Q
A

If an infarct includes the coronary blood supply to the papillary muscle(s) papillary valve rupture is likely to occur in the dead/damaged muscle. This would likely compromise cardiac output and blood pressure (afterload) would fall rather than rise.The jugular pulse would not be affected until congestive heart failure included the right heart.

The correct answer is: papillary muscle rupture.

How well did you know this?
1
Not at all
2
3
4
5
Perfectly
36
Q
A

The correct answer is: The musculature of the right ventricle

How well did you know this?
1
Not at all
2
3
4
5
Perfectly
37
Q
A

The correct answer is:

  1. A 73 year old man with a prior history of ischaemic heart disease is found dead in his flat. At autopsy there is evidence of scarring in the anterior 2/3 of the interventricular septum. → Left anterior descending artery,
  2. A 65 year old woman collapses in the mall and is unable to be resuscitated. At autopsy there is a cardiac tamponade with rupture of the lateral aspect of the left ventricular wall. → Left marginal artery,
  3. A 55 year old smoker collapses at the golf course. He had a history of episodes of angina. He could not be rescuscitated. At autopsy there were multiple areas of infarction in the right atrium and right ventricle. → Right coronary artery
How well did you know this?
1
Not at all
2
3
4
5
Perfectly
38
Q
A

A range of key observations: rate is normal and rhythm is stable (~60bpm) (no premature beats or other arrhythmias). Abnormal shape seen in all cycles, characteristic of established arterial disease and ischaemia. Inversion of the T-wave, no clear ST segment (ST elevation. Finally, the split of the R-wave in every cycle represents a conduction differences down right and left bundles of His and Purkinje systems. This implies some ischaemia in one side of the conduction system.

Note that the heart rate is ~1/sec = 60/min.

Fever- rheumatic or pericarditis (incorrect) causes tachycardia.

Hypothyroidism (incorrect) causes bradycardia.

The correct answer is: Coronary artery disease

How well did you know this?
1
Not at all
2
3
4
5
Perfectly
39
Q
A

Factor III (also known as Tissue Factor or Tissue thromboplastin) is a protein expressed in all cells except endothelial cells. When it is released in tissue damage, it combines with Factor VII and greatly accelerates coagulation by activating both Factor IX and Factor X.

The correct answer is: release of Factor III by the subendothelium.

How well did you know this?
1
Not at all
2
3
4
5
Perfectly
40
Q
A
  1. Individuals lacking Factor VIII are prone to excessive bleeding unless treated with concentrated solutions of recombinant Factor VIII. Genetic inheritance is X-linked recessive so affected individuals are males and the gene is carried by females.
  2. The cleavage of circulating fibrinogen to fibrin, catalysed by thrombin, allows for formation of the fibrin-rich secondary haemostatic plug. Thrombin also catalyses the activation of Factor XI, Factor VIII and Factor V as well as playing a role in platelet activation.
  3. The Vitamin K dependent clotting factors are those numbered II, VII, IX and X (2, 7, 9 & 10).

The correct answer is: 1. Inherited deficiency of this substance leads to a condition known as Haemophilia A → Factor VIII, 2. This substance plays an important role in the conversion of fibrinogen to fibrin and has positive feedback effects on several other stages of the coagulation cascade → Thrombin, 3. Oral anticoagulants which act as Vitamin K antagonists (e.g. warfarin) would reduce the amount of this substance → Factor X

How well did you know this?
1
Not at all
2
3
4
5
Perfectly
41
Q
A

The correct answer is: Myoglobin

How well did you know this?
1
Not at all
2
3
4
5
Perfectly
42
Q
A

The correct answer is:

  1. A 41 year man with a long history of alcohol abuse suffered prolonged bleeding after excision of a skin lesion. The bleeding last more than 30 minutes and was difficult to stop even with pressure. → Liver disease,
  2. A newborn premature baby who suffered an intracranial haemorrhage. → Vitamin K deficiency,
  3. A 55 year old woman with biliary obstruction due to a head of pancreas tumour → Vitamin K deficiency
How well did you know this?
1
Not at all
2
3
4
5
Perfectly
43
Q
A

The correct answer is:

  1. A 21 year woman presents complaining of a recent onset of easy bruising. On examination you find multiple petechial haemorrhages on her skin, especially on her lower legs and you notice some on the mucous membranes of her mouth. → Platelet count,
  2. A 65 year old man with atrial fibrillation (AF) who is on daily oral warfarin for stroke prevention attends the practice, as he was ‘a little slow’ to stop bleeding after he cut himself accidently in the kitchen. → Prothrombin time (PT),
  3. A two year old boy, with no family history of bleeding disorders, develops a painful, swollen right knee after a fall. No fracture is seen on X-ray and a diagnosis of haemarthrosis is given. In hospital he has a prolonged activated partial prothrombin time (APPT) but normal Prothrombin time (PT). → Factor VIII
How well did you know this?
1
Not at all
2
3
4
5
Perfectly
44
Q
A
  1. A split R-wave is indicative of a slowed conduction in one branch of the His-Purkinje system such that the muscle cells of one ventricle are activated and depolarise earlier the other, rather than the usual simultaneous activation
  2. A prolonged P-R interval is indicative of slowed conduction through the AV node. If too slow, the AV node may take over pace-making and the QRS complex occur regularly but will no longer be synchronised with each P wave
  3. An absent P wave but a regular QRS complex indicates failure of the S-A node to depolarise. The A_V node takes over as secondary pacemaker but at a slower than normal rate

The correct answer is: 1. The ECG record shows a persistent double peak or “split” in the R-wave → His – Purkinje system, 2. The QRS complex is separated from the P wave by a larger than normal P-R interval → Atrioventricular node, 3. A regular QRS complex occurs but the P wave is absent and R-R interval is prolonged and the heart rate slowed at rest → Sinoatrial node

How well did you know this?
1
Not at all
2
3
4
5
Perfectly
45
Q
A

The major controller of capillary exchange is vascular smooth muscle tone. Vascular smooth muscle will alter resistance, and therefore plasma hydrostatic pressure and velocity. Also contraction of the VSM can close some of the vessels and reduce surface area. a). altering capillary permeability; is not correct because permeability changes are most important in pathological states. b). altering lymphatic drainage; is not correct although this is a source of pathological variation in exchange.

The correct answer is: vascular smooth muscle contraction.

How well did you know this?
1
Not at all
2
3
4
5
Perfectly
46
Q
A

The correct answer is:
1. What is the cause of her sudden loss of consciousness? → Pooling of blood in the lower limbs
, 2. How does wiggling her toes or walking help? → Increased venous return to the heart,
3. What is the body’s normally response upon standing, that prevents light headedness? → Increased sympathetic nervous activity to the heart and blood vessels

How well did you know this?
1
Not at all
2
3
4
5
Perfectly
47
Q
A
  1. About 2/3 of all blood volume is in the systemic venous stimulation at rest
  2. Largest total cross sectional area and least total resistance due to so many parallel vessels results in lowest velocity of flow.
  3. Increased metabolic activity increases limb blood flow which might increase 8 fold or more during intense exercise. However, the pulmonary circulation takes the entire cardiac output and will always have greater blood flow than any other individual organ or organ system this will increase by a similar volume per minute as does the limb flow, this represents only a 2-3 fold increase in cardiac output, therefore, proportionally the pulmonary blood flow may not increase to the same extent as the limb blood flow. However, with a sudden unprepared response like this, the major contributor is the cardiac output. Metabolic takes some time to “catch up”.

The correct answer is:
1. In order to meet his increased metabolic requirements, a greater volume of blood must be directed to the heart to support increased cardiac output. Which part of the cardiovascular system contains the greatest proportion of the total blood volume at rest? → Systemic veins and venules,

  1. With an increase in cardiac output, mean velocity of blood flow will increase. Which part of the cardiovascular system has the lowest velocity of blood flow at rest? → Systemic capillaries,
  2. In association the sudden increase in cardiac output, which part of the cardiovascular system has the greatest proportional increase in volume of blood flow at this time? → Pulmonary arteries and veins
How well did you know this?
1
Not at all
2
3
4
5
Perfectly
48
Q
A

The correct answer is:

  1. Blood flow to individual organs is primarily determined by: → Vascular resistance, 2. Korotkoff sounds heard during auscultatory determination of blood pressure represent: → Blood turbulence,
  2. Coronary artery blood flow is largely driven by: → Diastolic blood pressure
How well did you know this?
1
Not at all
2
3
4
5
Perfectly
49
Q
A

The correct answer is:

  1. The resistance to blood flow is greatest in the → Arterioles,
  2. The velocity of blood flow is lowest in the: → Capillaries,
  3. The percent of volume of blood is highest in the: → Veins
How well did you know this?
1
Not at all
2
3
4
5
Perfectly
50
Q
A

The predominant adrenergic receptor type in the myocardium and pacemaker regions of the heart is the β1 adrenoceptor.

The correct answer is: Beta 1 adrenoreceptors

How well did you know this?
1
Not at all
2
3
4
5
Perfectly
51
Q
A

Cardio selective indicates it acts on heart (muscle and pacemaker) via the particular adrenergic receptors located in the heart. As the adrenergic receptors mediate positive inotropic and positive chronotropic actions of adrenaline and noradrenaline in the heart, a drug that blocks these receptors (competitive antagonist) will reduce the effectiveness of adrenaline and noradrenaline and therefore reduce force and rate of cardiac contraction. Note that a drug that reduces sympathetic neurotransmitter release would have a similar effect, however it can in no way be regarded as cardio selective as it would reduce sympathetic activity at all receptor types innervated by the sympathetic NS.

The correct answer is: Decreasing the rate and force of cardiac contractions

How well did you know this?
1
Not at all
2
3
4
5
Perfectly
52
Q
A

For patients without CVD we use a combination of CV event risk estimates and BP levels before deciding on treatment.

The correct answer is: Drug therapy for hypertension is based on a combination of: estimation of absolute cardiovascular risk; and BP thresholds for treatment

How well did you know this?
1
Not at all
2
3
4
5
Perfectly
53
Q

Look at the ambulatory BP monitoring result below for a patient whose BP was 152/106 in clinic and choose the statement that best describes the pattern of BP recorded.

A

A) mean daytime average 129/79 mmHg suggests good control so no need for extra treatment.
B) normal lower night-time BP “dipping” seen
C) not all BP readings are normal (e.g. max of 149/108)
D) no evidence of very low BP and would need patient diary for postural symptoms
E) correct answer … Clinic BP 152/106 and first reading at start of ABPM when patient at the hospital 149/108. BP then falls to average daytime of 123/79 mmHg.

The correct answer is: Possible white coat hypertension

How well did you know this?
1
Not at all
2
3
4
5
Perfectly
54
Q
A

The pericardium provides a restrictive sack that normally prevents over-distension of the ventricle.
The correct answer is: Fibrous pericardium

How well did you know this?
1
Not at all
2
3
4
5
Perfectly
55
Q
A
  1. Arteriolar resistance decreases under the influence of epinephrine (beta2 receptors) mediated vasodilatation in order to increase the delivery of oxygen and maintain oxygen concentration in the muscle. Arteriolar resistance may even decrease prior to exercise in preparation / anticipation of the increased oxygen demand.
  2. Coronary arteriolar resistance will also decrease during exercise leading to an increase in coronary vascular conductance. Lactic acid and CO2 production may increase but this will be found in the venous not the arteriolar circulation. Sympathetic NS activity increases with exercise but coronary arterioles are not innervated.
  3. Oxygen concentration drops in the central venous circulation due to extraction during passage through the exercising muscles.

The correct answer is: 1. Which of the following is most likely to decrease in her skeletal muscle vasculature during exercise? → Arteriolar resistance, 2. Which of the following is most likely to increase in the coronary arterial circulation during exercise? → Vascular conductance, 3. Which of the following is most likely to decrease in the central venous circulation during exercise? → Oxygen concentration

How well did you know this?
1
Not at all
2
3
4
5
Perfectly
56
Q
A

Atropine would block muscarinic receptors, inhibiting sweating and reducing skin blood flow and thus reduce heat-loss processes. Muscarinic block would also dry mouth secretions and inhibit vagal slowing of the heart.
A beta agonist would (like atropine) raise the heart rate but with little effect on the skin and would likely reduce blood pressure due to peripheral vasodilatation in muscle beds. It would stimulate metabolism (and therefore generate heat) but without inhibiting processes of heat loss. A beta blocker would tend to slow the heart and have little effect on the skin. It might slow metabolism and therefore reduce heat generation.
A non-steroidal anti-inflammatory (aspirin-like) drug will have no effect on cardiovascular parameters and unless Mr Wilson has an elevated body temperature due to fever, it will have no effect on skin blood flow or sweating.

The correct answer is: The patient has been given a drug to inhibit muscarinic receptors

How well did you know this?
1
Not at all
2
3
4
5
Perfectly
57
Q
A

In a case-control study design, investigators identify cases, patients who have already developed the outcome of interest (coronary artery disease) then choose controls, persons who do not have the outcome of interest, but who are otherwise similar to the cases with respect to important determinants of outcome such as age, sex and concurrent medical conditions. Investigators can then assess retrospectively the relative frequency of exposure to the putative harmful agent (high plasma cholesterol) among the cases and controls. The coronary artery disease patients represent cases and the disease free patients represent controls.

The correct answer is: Case-control study

How well did you know this?
1
Not at all
2
3
4
5
Perfectly
58
Q
A

In a randomised controlled trial, one can be most certain about what the subjects have done / taken that might have influenced their final outcome. It provides the best insurance that the result was due to the intervention.
The correct answer is: Randomised -controlled trial

How well did you know this?
1
Not at all
2
3
4
5
Perfectly
59
Q
A

Integrate your knowledge of basic principles of pressure and flow with causes of infarction. The presence of diabetes and vascular disease predicates poor circulation but localisation to the right leg suggests involvement of the knee brace restricting blood flow. Hypoxia or a low cardiac output would make this worse but the swelling in the limb indicates that blood is entering but not leaving the limb indicating external compression sufficient to block venous flow but not arterial flow (Venous infarction caused by venous compression). If arterial flow was blocked there would be no swelling (blood not entering or leaving). Tachycardia (fast heart rate) is unlikely if he has been sleeping all the time. Loss of creatine kinase from cells is a marker of muscle cell damage but not the cause.

The correct answer is: Venous infarction caused by venous compression

How well did you know this?
1
Not at all
2
3
4
5
Perfectly
60
Q
A

Coronary dominance refers to which coronary artery supplies the posterior interventricular artery, which in turn supplies the posterior 1/3 of the interventricular septum. In left-dominant coronary circulation, the circumflex artery supplies the posterior interventricular artery and the posterior 1/3 of the interventricular septum, whilst the left anterior descending artery still supplies the anterior 2/3. As these are both branches of the left coronary artery, this supplies the entire interventricular septum, which is important clinically given this is the most frequently blocked coronary artery.

The correct answer is: Entirely by his left coronary artery

How well did you know this?
1
Not at all
2
3
4
5
Perfectly
61
Q
A

Adrenaline and noradrenaline have similar potency and efficacy at vascular alpha-receptors (alpha1) and stimulate a similar vasoconstriction in the resistance vessels to increase total peripheral resistance and diastolic BP. However, Adrenaline has greater potency than noradrenaline at smooth muscle (vascular and bronchiolar) where it causes relaxation and this vascular relaxation (mainly in the large vessels supplying skeletal muscle will tend to reduce resistance and counteract the rise in diastolic BP (Answer c)). Answer a) A partial agonist could provide a similar contrasting effect (same potency but less maximum effect) but is not true for adrenaline. Answer b) is a correct statement but not the correct reason for effects on diastolic BP.

The correct answer is: Adrenaline has greater potency on vascular beta-receptors

How well did you know this?
1
Not at all
2
3
4
5
Perfectly
62
Q
A

The cyanosis (turned blue) reflects impaired oxygenation of the blood which is exacerbated by breath-hold (suckling, crying). Tetralogy of Fallot, is a congenital condition associated with ventricular septal defect, pulmonic valve stenosis, an overriding aorta, and right ventricular hypertrophy. The combination of pulmonary valve stenosis and ventricular septal defect, plus the overriding aorta will lead to right to left shunt and bypass of the pulmonary circulation. Initially asymptomatic because sufficient blood is oxygenated for basic (non-active) metabolism. Atrial septal defect, coarctation of the aorta and patent ductus arteriosus will all lead to left to right shunts and increased flow through the pulmonary circulation. This requires excess work by the heart and will lead to left ventricular hypertrophy. All conditions would be associated with dyspnoea (shortness of breath) as the child becomes more physically active.

See Lecture: Common congenital malformations of the heart.

The correct answer is: Tetralogy of Fallot

How well did you know this?
1
Not at all
2
3
4
5
Perfectly
63
Q
A

Beta-blocker (called antagonist on this occasion to check your understanding of agonism and antagonism), is the treatment of choice for angina and a suitable first-line drug for hypertension especially when angina is also present as it reduces cardiac work and oxygen consumption. A calcium channel blocker acting on the heart might have been a suitable alternative, but not one acting in the periphery.

The correct answer is: Beta adrenoceptor-antagonist

How well did you know this?
1
Not at all
2
3
4
5
Perfectly
64
Q
A

The correct answer is:
1. Richard is a 38 year old man who was found to be hypertensive on a routine medical. Whilst examining Richard you notice that he has very large hands, significant prognathism and prominent supraorbital ridge. → Acromegaly,

  1. Lily is a 34 year old woman who has been referred by the practice nurse as she has had three blood pressure readings all greater than 160/100. On examination you discover a bruit over the right side of her posterior chest wall / back around L1. → Renal artery stenosis,
  2. George is a 46 year old man who presented for a general ‘check up’. During the examination and on several occasions subsequently, he has been found to have high blood pressure. Examination reveals that he has dry skin, some pre-tibial oedema and slow relaxing reflexes. → Hypothyroidism
How well did you know this?
1
Not at all
2
3
4
5
Perfectly
65
Q
A

The correct answer is: Trachea

How well did you know this?
1
Not at all
2
3
4
5
Perfectly
66
Q
A

Blood flow (cardiac output) is equal for pulmonary and systemic circulation. However, the pulmonary circulation is a low pressure system with lower resistance.

The correct answer is: Lower resistance

How well did you know this?
1
Not at all
2
3
4
5
Perfectly
67
Q
A

With inadequate production of surfactant in the immature lung the alveoli have a greater surface tension and tend to decrease in size and collapse. Lung collapse would lead to a lower then normal PO2 and faster breathing rate. Pulmonary vascular resistance increases with low PO2 (unlike systemic resistance which decreases in response to low PO2).

The correct answer is: Collapse of the small alveoli

How well did you know this?
1
Not at all
2
3
4
5
Perfectly
68
Q
A

Ductus arteriosus connects pulmonary artery to aorta during foetal development to shunt blood past the high resistance of the unused pulmonary vasculature. It closes at birth. If it remains patent then blood will pass from the highest pressure (aorta) to a point of lower pressure (pulmonary artery)
The correct answer is: From aorta to pulmonary artery

How well did you know this?
1
Not at all
2
3
4
5
Perfectly
69
Q
A

The correct answer is: has a cardiac impression and an aortic groove on its mediastinal surface.

How well did you know this?
1
Not at all
2
3
4
5
Perfectly
70
Q
A

The major vessel supplying the leg is the popliteal artery. It runs in the popliteal fossa, just posterior to the knee joint and complete flexion of the knee compresses this artery.

The correct answer is: Popliteal artery

How well did you know this?
1
Not at all
2
3
4
5
Perfectly
71
Q
A

The genicular arteries form an anastomosis around the knee joint and are particularly useful during complete knee flexion as an alternate route for blood from the thigh to the leg. They anastomose on the anterior surface of the knee, therefore are not occluded during knee flexion. Veins do not provide an alternate route for delivery of arterial flow.

The correct answer is: Superior and inferior genicular arteries

How well did you know this?
1
Not at all
2
3
4
5
Perfectly
72
Q
A

The phases of development are in order; Embryonic, Pseudoglandular, Canalicular, Terminal sac, Alveolar. Terminal bronchi from during the pseudoglandular phase then branch into respiratory bronchi in the canalicular phase and form terminal sacks and primitive alveoli during the terminal sac phase before the final development of mature alveoli which form in the alveolar phase and continue to from for up to 3 years.

The correct answer is: Canalicular

How well did you know this?
1
Not at all
2
3
4
5
Perfectly
73
Q
A

Pulmonary hypoxic vasoconstriction caused by low PO2 in the alveoli occurs throughout the lung because of the low PO2 of the inspired air.

The correct answer is: Generalised pulmonary vasoconstriction

How well did you know this?
1
Not at all
2
3
4
5
Perfectly
74
Q
A

Increased PCO2 will cause vasodilation of the cerebral arterioles and decreased PCO2 will cause vasoconstriction arterioles. There is little change to brain blood flow over a large decrease PO2. There is little direct sympathetic innervation to cerebral arteriole (stimulation of sympathetic cervical ganglion as no effect on cerebral blood flow). Histamine and prostacylin are circulating metabolites and have little or no effect as they are unable to cross the blood brain barrier.

The correct answer is: Carbon dioxide

How well did you know this?
1
Not at all
2
3
4
5
Perfectly
75
Q
A

Skin has extensive sympathetic innervation and is under extrinsic control. All the other organs listed are primarily local metabolic factors. Skeletal muscle is under sympathetic extrinsic control when at rest (not contracting).

The correct answer is: Skin

How well did you know this?
1
Not at all
2
3
4
5
Perfectly
76
Q
A

Reactive hyperaemia = An increase in blood flow to an organ occurs after a period of no blood flow and is proportional to the occlusion period.

The correct answer is: Reactive hyperaemia

How well did you know this?
1
Not at all
2
3
4
5
Perfectly
77
Q
A
  1. Hyperpnoea is any increased depth and rate of breathing. Hyperventilation is increased rate or depth of breathing to abnormal levels such that the healthy balance between breathing in O2 and breathing out CO2 is upset, with more CO2 cleared from the lungs, causing decreased arterial concentration of CO2. Tachypnoea is abnormally rapid breathing, Apnoea means absence of breathing. Dyspnoea means difficult breathing. Also known as shortness of breath or breathlessness, dyspnoea is a subjective awareness of the sensation of uncomfortable breathing.
  2. Dyspnoea means difficult breathing. Also known as shortness of breath or breathlessness, dyspnoea is a subjective awareness of the sensation of uncomfortable breathing.
  3. Hypercapnia means an abnormally high arterial CO2 concertation and is the main stimulus for breathing. Hypercapnia develops when breathing is disturbed to the extent of restricting lung clearance of CO2 which in turn reduces the concentration gradient for CO2 diffusion from blood. Hypercapnia will cause to (respiratory) acidosis. Conversely, acidosis of metabolic origin will induce hyperventilation to drive off CO2 (respiratory compensation) and reduce arterial CO2 concentration.

The correct answer is:

  1. What is the term used to describe Julie’s increase in breathing rate during normal exercise, that matches her increased metabolic demand? → Hyperpnoea, 2. What is the term used to describe the feeling that she cannot get enough air into her lungs to satisfy her body’s oxygen needs during an asthma episode. → Dyspnoea,
  2. What is the stimulus for her increased breathing effort and rate during an asthma episode? → Hypercapnia
How well did you know this?
1
Not at all
2
3
4
5
Perfectly
78
Q
A

The correct answer is: Decreased CO2

How well did you know this?
1
Not at all
2
3
4
5
Perfectly
79
Q
A

The correct answer is: Glucose-6-phosphate dehydrogenase deficiency

80
Q
A
  1. Ankle oedema requires adequate arterial blood flow to provide the extravascular fluid retention. All other symptoms reflect impaired arterial blood flow.
  2. The first visible feature of acutely impaired blood flow would be pallor of the affected region. Ulceration and gangrene are long term consequences and pain on walking a distance would not be seen during visual inspection of the legs.
  3. Claudication is a term derived from the Latin word meaning “to limp”. Intermittent claudication describes the pain that develops in the muscles of the legs when taking exercise, such as walking. The claudication (limping) is intermittent because it only develops as muscle becomes hypoxic after walking some distance and would not be usually present, unlike limping associated with a leg or foot injury that is continuously evident.

The correct answer is:

  1. A 76-year-old woman has all of the typical symptoms of arterial disease of the lower limb, and no other symptoms. Which feature, from the list above, are you least likely to see? → Ankle oedema,
  2. A 62-year-old man with an abdominal aortic aneurysm suffers an acute arterial occlusion in his right leg as a result of embolisation of thrombus from the aneurysm. He is seen immediately. Which feature will be seen on visual inspection of the legs? → Pale discolouration of the skin,
  3. A vascular surgeon writes in the clinical notes of a 55-year-old woman that she has “intermittent claudication”. What feature did the patient describe? → Pain on walking a distance
81
Q
A

Approximately 60% of CO2 in blood is in the form of HCO3- (bicarbonate ion) with only 30% carried on haemoglobin, unlike O2 which is >98% carried via haemoglobin.

The correct answer is: bicarbonate ion.

82
Q
A

There is less haemoglobin, which would be fully saturated leaving a normal PO2, determined by the dissolved O2 only. The arterial oxygen content would be reduced by about 1/3 because of the 1/3 reduction in haemoglobin.

The correct answer is: Normal arterial PO2 but reduced arterial oxygen content of about 13mL/100mL

83
Q
A

Pleural effusion is an excessive accumulation of fluid in the pleural space which shows up as radio-opaque in the chest X-Ray.

Fluid will always gravitate to lower regions.

Pneumothorax represents air in the thoracic cavity and this is not radio-opaque. Lung collapse would show as volume loss of the right upper (answer d) or right lower lobe (answer b) and lobar collapse (with less air in the lung) does make the lung tissue appear more dense in the collapsed region. However, lower right lung collapse would be differentiated from right sided pleural effusion by the volume loss associated with the collapse on the right compared to the left hemi thorax displacing the trachea which would be PULLED to the right (not seen in this image).

The correct answer is: Right sided pleural effusion

84
Q
A

The correct answer is:

  1. What is the volume or capacity labelled number 3 in the diagram? → Tidal volume (VT),
  2. What is the volume or capacity labelled number 5 in the diagram? → Vital capacity (VC),
  3. What is the volume or capacity labelled number 6 in the diagram? → Residual volume (RV)
85
Q
A

Air can only enter the lungs (inspiration) if the pressure in the alveoli is lower than the pressure outside (atmospheric pressure). Contraction of the diaphragm creates negative pressure in the intrapleural space which in tern stretches the alveoli creating lower pressure in them with respect to atmosphere.

The correct answer is: Intrapleural pressure is more negative than during expiration

86
Q
A
Dyspnoea = breathlessness or difficulty in breathing
Hypercapnea = high blood CO2
Hyperpnea = rapid breathing
Hypoventilation = low ventilation of the lungs (alveoli)
Orthopnea = dyspnea that is relieved by sitting upright (ortho = position)

The correct answer is: Hypoventilation

87
Q
A

Blood flow is naturally greater in the lower parts of the lung due to hydrostatic pressure keeping the capillaries open. When lying on her right side, the right lung becomes the lower part of the lung.

The correct answer is: Higher blood flow per unit volume

88
Q
A

Carotid chemoreceptors detect O2 whereas medullary chemoreceptors detect H+ ion and the other receptors are mechanoreceptors detecting stretch.

The correct answer is: Carotid body chemoreceptors

89
Q
A

High oxygen levels will remove the hypoxic stimulus for breathing. If his breathing rate is slow then there is evidence that he is already desensitised to CO2. under these conditions Stephen is dependent upon his hypoxic response to stimulate breathing.

The correct answer is: Increase his arterial PO2 and further depress his breathing

90
Q
A
  1. Cough syncope is a well-recognised occurrence in patients with COPD or asthma. It occurs with paroxysms of coughing associated with vigorous muscular effort. Severe coughing increases thoracic pressure, opposing and therefore depressing venous return. Cardiac output decreases with a sudden drop in cerebral blood flow. It is actually uncommon in women because they are less likely to generate the very high pressures with coughing.
  2. Low blood haemoglobin after blood donation amounts to anaemia. Three days after the donation, blood (fluid) volume would be restored but red cell count would not.
  3. Hyperpnoea is any increased depth (and rate) of breathing, hypopnoea means small breaths and tachypnoea is abnormally rapid (often shallow) breathing. Apnoea means absence of breathing. Dyspnoea means difficult breathing. Also known as shortness of breath or breathlessness, dyspnoea is a subjective awareness of the sensation of uncomfortable breathing.

The correct answer is:

  1. The sudden blackout caused by her coughing would most likely be the result of: → Ischaemic hypoxia,
  2. Several months earlier, Vanessa had passed out three days after making a blood donation. That blackout would most likely be the result of: → Anaemic hypoxia,
  3. Following her rapidly regaining consciousness, she is found taking large, gasping breaths which would be described as: → Hyperpnoea
91
Q
A

The correct answer is: Increased cardiac output and more homogeneous ventilation-perfusion ratios throughout the lung.

92
Q
A

Smoke inhalation and its treatment provokes a complicated response. In this instance the patient has a metabolic acidosis caused by initial tissue hypoxia during the smoke exposure.

This is evident in the low pH which shifts the HbO2 curve to the right and aids Hb unloading of oxygen. The low CO2 indicates that the low pH is not respiratory in nature (it is probably due to a compensatory increased breathing rate stimulated by the low pH).

A low CO2 as primary effect would not stimulate breathing rate. Indeed it is caused by an elevated breathing rate.

A low O2 saturation will not stimulate elevated breathing rate unless PO2 was low as well.

A high PO2 has no direct effect on breathing rate

The correct answer is: Decreased pH aids tissue unloading of oxygen

93
Q
A

The correct answer is:

  1. Which represents collapse of the alveoli and small airways? → Atelectasis,
  2. Which represents inflammation of the terminal branches of the respiratory tree? → Bronchiolitis,
  3. Which represents the destruction of alveolar walls and permanent, pathological dilation of the bronchi? → Bronchiectasis
94
Q
A
  1. Working through step by step, the low pH indicates an acidosis, and the parameter most consistent with this is the low bicarbonate level therefore it is a metabolic acidosis. The low pCO2 is consistent with alkalosis, so this must be the respiratory compensation for the metabolic acidosis i.e. blowing off more CO2 to reduce the acid load in the body.
  2. hypoxemia stimulates peripheral chemoreceptors.
  3. This picture is an alkalosis (pH 7.47), and pCO2 being low is consistent with this, so it looks like a respiratory alkalosis. The oxygenation is normal i.e. 100 mmHg, indicating that there is sufficient perfusion (i.e. not a COPD situation or high altitude) so hysterical hyperventilation is the most likely explanation. The bicarbonate is a little lower than normal so the renal compensation may have started to kick in (depending on how long ago the patient hyperventilated) but is not fully compensated.

The acid-base imbalances caused by these conditions are as follows:

hyperaldosteronism – renal H+ loss leading to metabolic alkalosis
hysterical hyperventilation – resp. alkalosis
loop diuretics – metabolic alkalosis
barbiturate overdose – respiratory depression leading to respiratory acidosis
vomiting – metabolic alkalosis from loss of HCl
stroke – respiratory depression leading to respiratory acidosis
high altitude – respiratory alkalosis
chronic renal failure – metabolic acidosis
The correct answer is:
1. Which of the options given in the list would be most likely to cause the acid-base imbalance in this patient? → chronic renal failure,
2. Which of the options given in the list elicits its response via peripheral chemoreceptors? → high altitude,
3. She had been admitted to emergency only a week before and blood gas analysis at that time showed an arterial blood pH value of 7.47, a PCO2 of 25 mmHg, a PO2 of 100 mmHg and a bicarbonate concentration ([HCO3-]) of 20.2 mEq/L. Which of the options in the list would be most likely to cause this acid-base imbalance? → hysterical hyperventilation

95
Q
A

Respiratory alkalosis, uncompensated. Explanation: The [H+] is extremely low, therefore pH is high indicating an acute alkalosis. The primary cause is respiratory due to the low PCO2 and metabolic compensation has begun as the bicarbonate levels are also lower than normal but pH has not returned to within normal range so the acid-base imbalance has not been fully corrected by the renal compensation.

The correct answer is: Respiratory alkalosis, uncompensated

96
Q
A

The correct answer is: Peak expiratory flow rate (PEFR)

97
Q

Bradley Johnstone is a 35 year old male with a history of asthma since childhood. He exercises regularly, mainly by swimming. One spring day he decides to watch some mates from work play their cricket final in the local park.

  1. Before the match starts he is joking with his friends and accidentally aspirates some water from a drink bottle, causing him acute dyspnoea and sending him into a coughing fit. What is the most likely mediator of his acute dyspnoea?
    Choose…
  2. It is spring and there are many flowering grasses and other plants in the park. After only 30 min in the park he begins to wheeze and has some difficulty breathing. What is the most likely mediator of his acute dyspnea?
    Choose…
  3. He decides to go home and on arrival feels ok so neglects to take his asthma medication. Later in the afternoon he develops severe wheeze and dyspnoea and takes himself to the hospital emergency department where he is found to have a severely depressed FEV1 during a lung function test. What is the most likely mediator of the dyspnea and wheeze at this time?
    Choose…
A

The correct answer is:
. Before the match starts he is joking with his friends and accidentally aspirates some water from a drink bottle, causing him acute dyspnoea and sending him into a coughing fit. What is the most likely mediator of his acute dyspnoea? → Bronchospasm via parasympathetic (vagal) activity,
2. It is spring and there are many flowering grasses and other plants in the park. After only 30 min in the park he begins to wheeze and has some difficulty breathing. What is the most likely mediator of his acute dyspnea? → Bronchospasm via mast cell degranulation,
3. He decides to go home and on arrival feels ok so neglects to take his asthma medication. Later in the afternoon he develops severe wheeze and dyspnoea and takes himself to the hospital emergency department where he is found to have a severely depressed FEV1 during a lung function test. What is the most likely mediator of the dyspnea and wheeze at this time? → Vasodilation and oedema via arachidonic acid derived prostaglandins, leukotrienes and platelet activating factor (PAF)

98
Q
A

a) correct answer
b) would result in systemic concentrations significant enough to cause adrenal suppression
c) can kill patient as ventilatory drive depends on hypoxia when PaCO2 chronically elevated
d) muscarinic receptors;
e) selectivity “lost” at high doses

The correct answers are: Cromolyns inhibit mast cell degranulation, The anticholinergic drug ipratropium works at acetylcholine nicotinic receptors.

99
Q
A

The bronchial cartilage is important in keeping the trachea and bronchi patent for smooth air conduction to the respiratory bronchioles and alveoli.
The correct answer is: Keep the trachea and bronchi patent

100
Q
A

Chromoglycate stabilises mast cells and prevents allergen or cold or exercise induced release of bronchoconstrictor – histamine- to be used before sport. Other choices such as short-acting beta 2 agonist could also be used (but not on the list) or oral montelukast leukotriene receptor antagonist taken 2h prior to exercise (as needed). This patient does not need daily preventer medication as the circumstances are infrequent but predictable.

The correct answer is: Cromoglycate inhaler used as needed*

101
Q
A

The correct answer is: Systematic review with meta-analysis, Randomised Controlled Trial, Cohort study, Case report

102
Q
A

Because of the characteristics of the parietal pericardial tissue, the intact pericardial sac has a limited reserve volume, becoming relatively noncompliant when the heart volume is at the high end of the normal physiologic range. Pericardial restraint most commonly occurs when a change in heart size occurs rapidly and exceeds the acute capacity for the pericardium to grow and accommodate. This might happen for example with subacute mitral valve regurgitation (acute worsening of valve insufficiency) or pericardial effusion, and exhibits an increased pericardial influence on cardiac filling.

The correct answer is: Elevated pressure throughout diastole

103
Q
A

The correct answer is: Internal intercostals

104
Q
A

The correct answer is: 1. Which represents collapse of the alveoli and small airways? → Atelectasis, 2. Which represents inflammation of the terminal branches of the respiratory tree? → Bronchiolitis, 3. Which represents the destruction of alveolar walls and permanent, pathological dilation of the bronchi? → Bronchiectasis

105
Q
A

The disease described is cystic fibrosis. Of the methods of choice, only PCR and DNA sequencing are able to identify small mutations in the CFTR gene. It would be important to compare blood samples from the woman, her sister and the affected child. Cytogenetic chromosome analysis (B) would only reveal large chromosome aberrations, such as Down’s Syndrome or chromosome translocations, whereas these are point mutations that need to be identified at the moleculer level. Whilst IRT (C) and the sweat test (E) are performed on potentially affected individuals, they are not useful to identify heterozygote carriers of the mutation. The same applies to chest x-rays (A).

The correct answer is: PCR analysis and DNA sequencing

106
Q
A

Enhanced sweat chloride concentration is a hallmark of cystic fibrosis. Of the listed organs only the lungs are affected. Another one would be the pancreas, (not listed here). There may be some complications with the intestine as secondary effects of the impaired exocrine function of the pancreas.
The correct answer is: Lungs

107
Q
A

The maxillary sinuses are in the maxillary bone (cheek area), so inflammation of these sinuses causes headache as well as toothache, as the superior alveolar process is in the maxillary bone, just inferior to the sinus.

The correct answer is: Maxillary sinus

108
Q
A

The “before” test is indicative of severely impaired lung function characteristic of obstructive disease with impaired peak expiratory flow due to exaggerated dynamic airway compression during effort and concave (non-linear) expiratory flow curve that would indicate impaired elasticity during the effort-independent limb of the curve. The lung function is improved in the “after” test with increased peak flow, some improved elasticity and restored expiratory volume. This is indicative of the ability of a dilator to acutely open the airways.

The correct answer is: Improved lung function after bronchodilator treatment

109
Q
A
  1. The yeast Candida albicans is normally present in the upper respiratory tract as part of the commensal flora. However, under certain conditions, such as treatment with broad spectrum antibiotics, it may overgrow other organisms and may cause candidiasis (oral thrush). Epstein–Barr virus, although it is present in about 90% of the ‘western’ population and although it may cause infectious mononucleosis, is not part of the normal flora; it exists intracellularly.
  2. Antigenic shift is a sudden major change in antigenic properties of viruses, only seen in influenza virus A. This enables the virus to infect people immunised against previous viral strains.
  3. Legionella bacteria thrive in warm water and warm damp places. Legionnaires’ disease is most readily acquired by breathing water vapour or fine water spray from an infected source.

The correct answer is: 1. Which agent is commonly present in the upper respiratory tract as part of the normal flora, but must be considered potentially pathogenic? → Candida albicans, The phenomenon of antigenic shift leads to alteration of surface molecules in this infectious agent. This shift is one of its strategies to evade attacks by the immune system. → Influenza virus, 3. Which infectious agent would be suspected as the cause for a disease outbreak linked to an aquarium or a car wash? → Legionella pneumophila

110
Q
A

ARDS occurs in critically ill or critically injured patients characterised by widespread inflammation in the lungs with diffuse alveolar damage and fluid accumulation. It is a clinical condition (syndrome) initiated by numerous pathologies: severe pneumonia; sepsis; near drowning; aspiration; inhalation injury. It can be triggered within 2 hrs of exposure but often takes 1-3 days to develop. No infection and afebrile indicates not pneumonia. There is no evidence of aspiration. Ateleactesis is the collapse of alveoli, usually in a portion of the lung, most commonly after surgery (anaesthetic effect) but also acutely with inhaled foreign object, mucous plug formation or chronic infections or tumour leading to permanent narrowing of the airways. Bronciolitis is a viral lung infection of the lower airways (bronchioles) leading to airway narrowing and mucous plugging, usually very young children (under 6 months or maybe up to 12 months).

The correct answer is: Acute respiratory distress syndrome (ARDS)

111
Q
A

Only spirometry will identify lung function characteristic of obstructive disease. The other tests will demonstrate the presence of acute or chronic conditions or diseases that may or may not be components of, or contributors to COPD signs and symptoms, but are not diagnostic of COPD.

The correct answer is: Spirometry lung function test

112
Q
A

Increased air flow resistance derives from the narrowing of small airways due to loss of elastic support and swelling from chronic inflammation. Lung compliance (stretch) is increased but rebound (elasticity) is decreased. The enlarged airspaces are the result of destruction of alveolar walls, with joining of adjacent alveoli into larger units but loss of total gas exchange surface area. Inspiratory volumes are reduced due to air trapping and retention of greater residual volume at start if inspiration. Despite smaller inspiratory volumes the initial over-expansion of the lungs, reduced lung elasticity and increased airflow resistance all conspire to reduce expiratory flow-rate and prolong expiration time.

The correct answer is: increased air flow resistance.

113
Q
A

Mitral valve stenosis: Mid diastolic murmur suggests a defect that is evident during filling (diastole). The location of the sound (aligned with the apex of the heart) suggests that it is the mitral valve. Mitral valve incompetence (incorrect) would manifest as a defect during ejection. Symptoms suggest heart failure and this is likely caused by the valve defect.

The correct answer is: Mitral valve stenosis

114
Q
A

The correct answer is: resistant to isoniazid and rifampin.

115
Q
A

The long history of cough, lack of immunisation against pertussis infection (whooping cough), and childhood illness suggests lung damage and bronchiectasis. TB is possible but much less likely. Had she had it as a child, at her age, it should have been treated effectively. Cystic fibrosis is also less likely with this history.

The correct answer is: Bronchiectasis

116
Q
A

The overall strategy should be to use a combination of drugs which operate by quite different mechanisms of action. This considerably reduces the likelihood of selecting for resistance mutations. Answers D and E can be excluded, since they show only two antibiotics each. Ampicillin and trimethoprim are not active against M. tuberculosis. Ciprofloxacin (an inhibitor of DNA topoisomerase) is considered as a second-line drug for Tb. This excludes answers A and B. C is the correct answer – refer to section 2 of the Tb & multidrug resistance GOAL.

The correct answer is: Isoniazid, rifampin, ethambutol and pyrazinamide

117
Q
A

The correct answer is: Is habitually present in human populations

118
Q
A

A sudden explosive increase in cases suggests a single exposure to a common vehicle. A common vehicle suggests that it is not transferred from one person to another (eg. Legionnaire’s disease outbreak), and hence there would not be secondary cases. Although the exposure is limited to one geographical area, the cases will be identified in many and varied places unless all exposed were restricted to a common area.
The correct answer is: Explosive increase in cases

119
Q
A

About 80% of pulmonary emboli have no clinical consequences.

The correct answer is: No clinical features

120
Q
A

Pleurisy is pleural inflammation. The fibrinous exudate of the inflammatory process causes the visceral and parietal pleura to stick to each other, resulting in an audible pleural rub.

The correct answer is: Pleurisy

121
Q
A

A biological gradient is another term for a dose response relationship. We are more confident attributing a harmful effect to a particular exposure if, as the quantity or the duration of exposure to the putative harmful agent increases, risk of the harmful effect also increases. It is not identifying a mechanism of action, nor is it identifying a relationship in time. There is evidence of two or more studies showing the same effect but this does not relate to the biological gradient. The occurrence in Zutphen identifies a location for the exposure but other studies show it also occurs elsewhere and this information is not related to biological gradient.

The correct answer is: There is a relationship between amount of exposure and extent of harm

122
Q
A

The anterior and middle ethmoidal sinuses, frontal sinus and maxillary sinus all open into the middle meatus. The sphenopalatine foramen, posterior ethmoidal sinus and sphenoidal sinus open into the superior meatus. The nasolacrimal canal and incisive canal open into the inferior meatus.
The correct answer is: Anterior and middle ethmoidal sinuses and maxillary sinus

123
Q
A

The cricothyroid muscle stretches and tenses the vocal folds and enables production of high pitch vocal sounds.
The correct answer is: Cricothyroid

124
Q
A

The infrahyoid muscles lie below, but attach to, the hyoid bone and they depress the hyoid bone and larynx during speaking and swallowing.

The correct answer is: Depress the hyoid bone and larynx

125
Q
A

Bronchial carcinoids arise in the central airways from the Kulchitsky cells (neuroendocrine cells) that line the bronchial mucosa. They appear at an early age (mean 40 years) and represent about 5% of all pulmonary neoplasms. Carcinoids are often resectable and curable. They grow in one of two patterns: (1) an obstructing polypoid, spherical, intraluminal mass or (2) a mucosal plaque penetrating the bronchial wall to fan out in the peribronchial tissue—the so-called collar-button lesion. Histologically, these neoplasms, like their counterparts in the intestinal tract, are composed of nests of uniform cells that have regular round nuclei with “salt-and-pepper” chromatin, absent or rare mitoses, and little pleomorphism and dense core secretory granules on electron microscopy.
The correct answer is: Demonstrate salt-and-pepper chromatin and dense-core secretory granules

126
Q
A

This is because, virtually all SCLCs have metastasized by the time of diagnosis and hence are not curable by surgery. Therefore, they are best treated by chemotherapy, with or without radiation. In contrast, NSCLCs usually respond poorly to chemotherapy and are better treated by surgery.
The correct answer is: Small cell and Non-small cell carcinoma

127
Q
A

The correct answer is: Pregnancy

128
Q
A

The correct answer is: Murray Valley Encephalitis

129
Q
A

Statistics show that while patients with angina or pulmonary embolism are about twice as likely to be hospitalised if they also have COPD, the risk of heart failure resulting in hospitalisation is increased 4 fold.

The correct answer is: Congestive heart failure

130
Q
A

The high risk of premature cardiovascular disease death is restored to similar to non smokers within about 5 years. The risk of lung cancer begins to decrease immediately too but this occurs much slower and remains elevated for much longer.

The correct answer is: stays elevated for longer than the risk of cardiovascular disease mortality.

131
Q
A

The correct answer is: The Public Health Unit

132
Q
A

Mesothelioma is a rare but often fatal cancer principally due to asbestos fibre inhalation. Despite the recent reduction in industrial and environmental exposure, the long delay between exposure and symptomatic disease means that its incidence is still increasing. It is not related to the still increasing (or stable) rate of cigarette smoking in young women nor any prior respiratory disorders or associated conditions.

The correct answer is: amongst the highest in the world.

133
Q
A

The correct answer is: 1. He begins to pull his upper limb back through the glass but catches the anterior aspect of his arm, just superficial to Biceps Brachii, on the broken glass. Blood starts to ooze from the cut. Which superficial vessel might be cut? → Cephalic vein, 2. Jerking away from the pain, he accidentally stabs his forearm on another piece of glass, just proximal to the wrist on the medial aspect. Blood begins to flow freely from the puncture wound. Which vessel has been punctured? → Ulna artery, 3. Concerned about his condition, his flatmates take him to the emergency ward at the local hospital. As well as attending to the bleeding in the biceps and wrist, the nurse treats the cuts on Barry’s hand, paying particular attention to a large laceration on the back of his hand, running from digiti minimi to the indices. What vessel/s might be cut? → Dorsal metacarpal artery

134
Q
A

Because of the characteristics of the parietal pericardial tissue, the intact pericardial sac has a limited reserve volume, becoming relatively noncompliant when the heart volume is at the high end of the normal physiologic range. Pericardial restraint most commonly occurs when a change in heart size occurs rapidly and exceeds the acute capacity for the pericardium to grow and accommodate. This might happen for example with subacute mitral valve regurgitation (acute worsening of valve insufficiency) or pericardial effusion, and exhibits an increased pericardial influence on cardiac filling.

The correct answer is: Elevated pressure throughout diastole

135
Q
A
  1. The hyperventilation increases removal of CO2 from the lungs (blows off CO2), decreasing alveolar PCO2 and increasing trans-alveolar pressure difference and thereby increasing rate of CO2 removal from the circulation and a fall in arterial PCO2. This removes the primary stimulus to breathe.
  2. At 10m depth, the water exerts approximately 2x the pressure at surface level and the alveolar PO2 (dissolved O2 ) is twice what it would be at the surface, increasing trans-alveolar pressure difference and increasing arterial PO2 by up to 2x.
  3. Hyperventilation artificially lowers blood CO2; metabolism depletes arterial O2 but with CO2 initially very low, the increased metabolically derived CO2 does not raise PCO2 sufficiently to reach breathing trigger levels but PO2 rapidly decreases as the external pressure declines from 2atm to 1atm and the swimmer loses consciousness. Unconsciousness forces a breath and drowning.

The correct answer is: 1. In order to stay under for longer, he hyperventilates prior to diving in. How does this enable him to hold his breath for longer? → Reduced arterial PCO 2 and decreased stimulation of central chemoreceptors, 2. The waterhole is over 10m deep and near the bottom he is able to continue to maintain a high arterial PO2 despite having held his breath for almost 30secs and swimming hard. What is the basis of the high arterial oxgen partial pressure? → Abnormally high alveolar PO 2, 3. He makes it to the bottom of the waterhole stopping to find a pebble to take from the bottom as evidence. On his ascent, as he nears the surface, he passes out and has to be rescued from the water and resuscitated. Attending ambulance officers tell him he has probably suffered from a “shallow water blackout”. What is responsible for this phenomenon? → Hypoxaemia coupled with low stimulation of central chemoreceptors

136
Q
A

Circumstances suggest acute exposure to irritant in his sleeping quarters which is greater on weekends when exposure is greatest. Pleural effusion, pulmonary congestion, ventilation perfusion mismatch would be expected to be more continuously present and observable on examination.

The correct answer is: Bronchospasm

137
Q
A

The rate ratios tell us that with respect to ischaemic heart disease, male and female indigenous Australians have 7.2 and 16.2 times the death rate respectively than non indigenous males and females. Whilst smoking might be more prevalent and be a major contributor to ischaemic heart disease deaths, this cannot be determined from the rate ratios or other data provided here. Despite the higher rate ratio for females than males the ratio is a comparison between indigenous and non-indigenous and the mortality rate data shows less females die of ischaemic heart disease than males (indigenous or non-indigenous). Answer C is incorrect because the rate per 100,000 doesn’t allow us to tell us the actual number of deaths. In fact because of the lower indigenous population, the absolute number of deaths is less.

The correct answer is: Non-indigenous Australians have a lower incidence of death from ischaemic heart disease.

138
Q
A

Drugs are generally not used in this age group. Oxygen is administered to overcome hypoxia and oxygen saturated is monitored with satisfactory oxygen saturation breathing normal room air being a key criterion for discharge (along with parental satisfaction and normal feeding behaviour).
The correct answer is: Oxygen

139
Q
A

Thoracodorsal and circumflex scapula are branches of the subscapular artery and contribute to the scapula anastomoses. The suprascapular and dorsal scapular arteries also contribute to the scapular anastomoses; however, they are branches of the thyrocervical trunk and thus, flow through these vessels would be impaired with partial arterial occlusion of the thyrocervical trunk.

The correct answer is: Thoracodorsal and circumflex scapular arteries

140
Q
A

The correct answer is: Mycotic

141
Q
A

The correct answer is: Median cubital

142
Q
A

If the brachial artery proximal to deep brachial branch is blocked, all pulses distal to this point (brachial at cubital fossa, radial and ulnar) will be absent. If the radial pulse is weak, this demonstrates flow down this vessel, which must be coming from the collateral circulation given that the ulnar pulse is absent. This means the blockage must be distal to the radial collateral branch which is a branch of deep brachial artery. Because the ulnar pulse is absent, the blockage must also be proximal to the ulnar collateral branches, of which, superior is the most proximal.

The correct answer is: Brachial artery distal to deep brachial artery, proximal to superior ulnar collateral branch

143
Q
A

With increased afterload associated with elevated arterial blood pressure, resistance to ejection will incur more of the cardiac cycle in developing pressure and less time for ejection. This will decrease the stroke volume, increase the end diastolic volume and decrease the left ventricular ejection fraction (LVEF). In work and energy terms, this will increase the potential energy component and the stroke work will represent a smaller proportion of the entire energy area, indicating a reduced efficiency of O2 use. ( a small adaptive increase in end diastolic volume will likely occur but this will be less prominent. This patient is at an increased risk of myocardial ischaemia.

The correct answer is: Decreased cardiac efficiency

144
Q
A

The correct answer is:
1. Whilst waiting for the ambulance, his flatmates apply a tourniquet to his upper arm to stem the excessive bleeding. On momentarily releasing the tourniquet after 5 minutes, they rapidly reapply it because the bleeding seems to be worse than it had been initially. What condition would most likely lead to this increase in blood flow to the cut arm? → Reactive hyperaemia,

  1. On admission to the hospital emergency department, he is made to lie down because he looks pale and feels faint. Barry reports that he has often felt “faint” over the past several months, usually when he stands suddenly after a period of lying down and resting. Prior to his injury, what condition might he have been experiencing? → Postural hypotension,
  2. One week later, Barry is readmitted after collapsing at home. On examination he is found to have a rapid pulse, systolic blood pressure of 85mmHg and difficult to detect diastolic pressure, rapid breathing and a body temperature of 40.2OC. What condition is he most likely to be experiencing? → Septic shock
145
Q
A

While low ratio of body fat to muscle; supine body position (due to improved venous return versus standing or sitting); young age (due to low body weight to surface area ratio requiring more energy to keep warm; and elevated resting metabolic rate due to growth); and Male gender (because of lean body mass to fat ratio but only after puberty); are all contributors to the cardiac output, only the fever (stimulating metabolic rate and therefore whole body oxygen demand requiring greater cardiac output to deliver the extra oxygen throughout the body) represents a cause of unusually high cardiac output.

The correct answer is: Whole body metabolic oxygen demand

146
Q
A

The correct answer is:
1. What occurs in the heart at the time corresponding to point 2 on the diagram? → Aortic valve opening,

  1. What occurs in the heart between point 4 and point 1? → Ventricular filling,
  2. What does the first heart sound correspond to? → Mitral valve closing
147
Q
A
  1. Work = force x distance. Stroke work = the area bounded by the pressure volume loop and represents developed pressure (force) x stroke volume (distance).
  2. Cardiac output = HR x Stroke volume (stroke volume can be determined from the X-axis)
  3. Stroke work is increased in loop 4 but as a consequence of the increased contractility not a cause. Increased contractility ensures a more forceful contraction which reaches the afterload pressure earlier and leaves more time for ejection and a more complete ejection.

The correct answer is:
1. What measure of heart function is demonstrated by the area bounded by each one of the loops? → Stroke work,

  1. What additional information, not obtainable from the diagram, is required in order to determine cardiac output for any of the conditions displayed? → Heart rate,
  2. Compared to the pressure-volume loop for the normal heart (numbered 1), what is most likely altered in the heart illustrated by pressure-volume loop 4 to cause the observed increase in systolic pressure? → Contractility
148
Q
A

Aortic valve closure creates the turbulence/ pressure wave that is detected as the second heart sound.

The correct answer is: Is party responsible for making the second heart sound

149
Q
A

The correct answer is: is formed where the great cerebral vein joins with the inferior sagittal sinus.

150
Q
A

The correct answer is: Posterior communicating artery

151
Q
A

The correct answer is:

  1. Rupert is diagnosed with dilated cardiomyopathy (DCM). An increase in which of the measures in the list is most indicative of this condition? → End diastolic volume,
  2. An increase in which measure provides the stimulus for the release of B-type natriuretic peptide? → End diastolic volume,
  3. From the data provided, which indicator of Rupert’s cardiac function is most clearly reduced? → Ejection fraction
152
Q
A

The correct answer is: Activation of the sympathetic nervous system

153
Q
A

The correct answer is: Anterior ethmoidal veins, ophthalmic veins, cavernous sinus

154
Q
A

The correct answer is:
1. Do you wake up during the 8 hours? How long does it take before you fall asleep? Do you find yourself waking up earlier than you plan to? → Insomnia,

  1. How long have you been feeling tired? Are you socializing as much as you were before you started feeling tired? Have you had any thoughts about harming yourself? → Hypersomnia,
  2. Do you drink alcohol before you go to bed? Do you exercise vigorously close to bed time? Are there any substances other than alcohol that you use? → CNS over-stimulation
155
Q
A

Any time the cardiac silhouette occupies more than 50% of chest diameter it is probably hypertrophied. All other answers represent common signs of pathology or disease.
The correct answer is: The cardiac silhouette occupies less than 50% of the chest diameter

156
Q
A

Inflation of the balloon wedges the catheter in a pulmonary small artery and isolates the tip from the pressure of the pulmonary artery behind it and creates a continuous link through the pulmonary arterioles, capillaries and veins to the left atrium. It measures “pulmonary wedge pressure”. It represents the left ventricular filling pressure
The correct answer is: Left atrial pressure

157
Q
A
  1. 2nd heart sound corresponds to closure of the aortic valve. This occurs when the ventricle has finished ejection (lowest volume) but pressure is just below aortic pressure (forces closure)
  2. Point C (incorrect) represents opening of the aortic valve. Although it represents the commencement of ejection, the volume has not yet decreased so no ejection has occurred yet. The P-V loop flows counter-clockwise. At point D the volume is decreasing (ejection).

3 Isovolumic means “same volume” and represents a period when the volume is unchanging as at F and B. At B (incorrect) the pressure is increasing (contraction), whereas at F the pressure is decreasing (relaxation).

The correct answer is: 1. Which point on the diagram corresponds to the second heart sound? → Point E, 2. Which point on the diagram corresponds to ejection of blood from the ventricle? → Point D, 3. Which point on the diagram corresponds to the isovolumetric relaxation of the ventricle? → Point F

158
Q
A

Most likely in this age group to be a vasovagal syncope caused by paradoxical activation of the vagus in response to reduced venous return, resulting in reduced cardiac output and TLOC (temporary loss of consciousness). Some minor jerking of limbs may appear to be like grand mal epilepsy but lacks tonic clonic features.

The correct answer is: Paradoxical vagal activity

159
Q
A

The cavernous sinus provides a means for communication between the facial vein (via the deep facial vein and pterygoid venous plexus) and the ophthalmic veins and thus the opportunity for superficial infections to spread to deeper areas via this route.

The correct answer is: Cavernous sinus

160
Q
A

The inferior alveolar artery is a descending branch of the mandibular portion of the maxillary artery and supplies the roots and pulp of the inferior teeth.

The correct answer is: Inferior alveolar artery

161
Q
A

Cholesterol is a risk factor, not a marker of disease, so the incidence of any manifestation of heart disease is not necessarily lower. The measurement of a risk factor is not a marker of burden of disease. The risk is lowered for only those individuals whose blood cholesterol is lowered (not everyone in the population), however, across the entire population the risk is reduced, so the population attributable risk is lower, therefore predicting a lower incidence in the future.

The correct answer is: Predicted decrease in the incidence of ischaemic heart disease in this population

162
Q
A

The correct answer is: It gives a good indication of right heart pressures

163
Q
A

Heart rate is determined by the influence of autonomic nervous system balance (parasympathetic and sympathetic) on the underlying sinus pacemaker rate. The transplanted heart is devoid of neural input.

  1. At rest, the main autonomic influence is elevated parasympathetic NS activity which slows the heart from its inherent pacemaker rate. Removal of the parasympathetic NS influence will reveal that higher (unbraked) pacemaker rate.
  2. Although the transplanted heart is devoid of neural input, the remainder of the sympathetic NS is intact and the exercise and emotional responses can still be mediated via the adrenal release of adrenaline.
  3. Exercise training induces some cardiac hypertrophy and increased metabolic efficiency. The physiological cardiac hypertrophy provides an adaptive increase in end diastolic volume that, through the Frank–Starling relationship induces a more powerful contraction and increased stroke volume. As further adaptation (preventing the increased stroke volume leading to increased arterial pressure), cardiac parasympathetic activity is increased at rest and cardiac output remains low at rest.

The correct answer is:
1.
George most likely has a high resting heart rate because, as a result of his transplant he is left with: → Decreased cardiac parasympathetic (vagal) nervous activity,

2.
What can George’s increase in heart rate during exercise most likely be attributed to? → Increased sympathetic adrenal activity,

3.
As a younger man, George had been quite a strong athlete with a low resting heart rate of less than 45bpm. What was the most likely basis for this low heart rate? → Increased cardiac parasympathetic (vagal) nervous activity

164
Q
A

Mitral valve stenosis: Mid diastolic murmur suggests a defect that is evident during filling (diastole). The location of the sound (aligned with the apex of the heart) suggests that it is the mitral valve. Mitral valve incompetence (incorrect) would manifest as a defect during ejection. Symptoms suggest heart failure and this is likely caused by the valve defect.

The correct answer is: Mitral valve stenosis

165
Q
A

If an infarct includes the coronary blood supply to the papillary muscle(s) papillary valve rupture is likely to occur in the dead/damaged muscle. This would likely compromise cardiac output and blood pressure (afterload) would fall rather than rise.The jugular pulse would not be affected until congestive heart failure included the right heart.

The correct answer is: papillary muscle rupture.

166
Q
A

The correct answer is: ACE (angiotensin converting enzyme) inhibitors block the conversion of angiotensinogen to angiotensin I, and this is the main substrate of the angiotensin receptor at the tissue level.

167
Q
A

The elevated body temperature and the likelihood of an infection arising from his earlier accident underpin this diagnosis of septic shock. There is no evidence of a severe allergic reaction which would include bronchospasm and urticaria (anaphylactic shock). Hypovolaemic shock due to excessive bleeding would result in cool clammy skin. Neurogenic shock would include slow heart rate and low body temperature. Cardiogenic shock usually arises from an acute myocardial infarction in the absence of existing heart failure and would include symptoms of pulmonary congestion.
The correct answer is: Septic shock

168
Q
A

Mitral regurgitation will produce left ventricular failure, which can then produce pulmonary hypertension and right ventricular failure. In this case, both ventricles will be abnormal. The other conditions will produce right ventricular failure without left ventricular failure; the result in these cases will be cor pulmonale (right ventricle enlarged but left ventricle normal).

The correct answer is: Mitral regurgitation

169
Q
A

ANP and BNP are released from “overstretched” atria and ventricles in response to volume overload and act in the kidney to increase glomerular filtration (renal dilatation) and reduce Na+ reabsorption in the collecting ducts. Water is retained in the collecting ducts in association with the Na+. The renin-angiotensin-aldosterone system and sympathetic nervous system serve to reduce renal blood flow/ glomerular filtration and re-absorb Na+ in the kidneys (water retention) whereas ADH stimulates water reabsorption directly. Vagal tone has a role in bladder but not renal function. Unfortunately, in heart failure, the compensatory responses to poor cardiac output and low blood pressure include counterproductive retention of fluid in effort to maintain blood pressure.

The correct answer is: Release of cardiac natriuretic peptides (ANP, BNP)

170
Q
A

Poor wound healing and infection risk are consequences of both smoking and diabetes due to suppressed immune response. Risk attributable to smoking can be diminished by stopping smoking. Nicotine in cigarette smoke induces tachycardia and high blood pressure due to stimulation of sympathetic ganglia, Carbon monoxide will impair optical measurement of oxygen saturation but not the measurement of oxygen partial pressures in the blood. Smokers have a high risk of respiratory impairment during and after anaesthetic but anaesthesia itself is not necessarily impaired. Diabetes may contribute to a reduced pain sensitivity due to peripheral neuropathy.

The correct answer is: poor wound healing and increased potential for infection.

171
Q
A

Calcium channel blockade will cause relaxation of smooth muscle and reduce cardiac force of contraction and conduction velocity. Relaxation of the vascular smooth muscle will impair reflex responses to posture change. Smooth muscle relaxation in the GI tract will cause constipation. Verapamil is most likely prescribed to the patient because of supraventricular tachycardia and if effective will reduce likelihood of stroke.

The correct answer is: Postural hypotension

172
Q
A

Exercise training improves muscle mitochondrial function and antioxidant activityto increase muscle oxygen efficiency, reducing oxygen demand (decreasing the cardiac output requirements of the heart) and increasing exercise reserve. (Myocardial efficiency is improved similarly, making it less prone to ischaemia.) Glucose uptake is improved but by increasing insulin sensitivity not insulin secretion.

The correct answer is: efficiency of muscle oxygen use.

173
Q
A

Endothelial damage provides the stimulus for the lipid deposition that is atherosclerosis.

Cytokines and chemokines are the cellular attractants produced during inflammatory responses

C-reactive protein is an inflammatory marker that rises as an early sign of inflammation.

The correct answer is: a) Atherosclerosis is initiated by: → endothelial damage., b) Cells are attracted to the atherosclerotic site due to: → cytokines & chemokines., c) Measurement of this parameter can be used in the assessment of risk of atherosclerosis: → C-reactive protein.

174
Q
A

The correct answer is:

a) A 45-year-old man, who has smoked 25 cigarettes per day for 20 years, underwent a total knee reconstruction a week ago. On the third post-operative day, he suddenly develops dyspnoea followed by chest pain in the left lateral part of his chest. The pain does not radiate and is exacerbated by deep inspiration and by coughing. His respiratory examination is normal, with the exception of tachypnoea (20 breaths/min). → Pulmonary embolism,
b) A 52-year-old woman presents to her family doctor with chest pain that has been occurring regularly while she is walking home over the past six months. She has a history of hypertension and hypercholesterolaemia, but has otherwise been well. While walking home she has to climb a hill, the pain comes on about half way up this hill. It is sometimes brought on by climbing stairs but not by anything else, and it settles after she rests for a few minutes. → Angina pectoris,
c) A 17-year-old adolescent, who is usually very fit because he plays basketball, is taken to the Emergency Department by his mother, after coming home from school early because of severe right-sided chest pain. It came on suddenly while he was sitting in class, and it hurts more when he takes a deep breath. He has felt slightly short of breath, but otherwise feels well. → Pneumothorax

175
Q
A

The correct answer is:

a) A patient reports she experienced dyspnoea and increased breathing rate during a recent high-trek in the Himalayas. → Hypoxaemia stimulating carotid body and aortic body chemoreceptors,
b) A patient hyperventilates for several seconds and experiences a prolonged period of apnoea before normal breathing resumes. → Hypocapnia and decreased stimulation of central chemoreceptors,
c) After a prolonged period of vomiting, a patient develops metabolic alkalosis and exhibits slowed breathing. → Reduced H+, hypocapnia and reduced stimulation of carotid and aortic body chemoreceptors

176
Q
A

The correct answer is:

a) The generation of this platelet activating factor is inhibited by aspirin. → Thromboxane A2,
b) Collagen is exposed by dislodgement of endothelial cells at the sites of trauma and bleeding. What substance plays a critical role in the binding of platelets to the exposed collagen fibers in the early stages of haemostasis? → von Willibrand factor,
c) A deficiency of Vitamin K (e.g. from malabsorption) would most likely affect the synthesis of which factor by the liver? → Factor VII

177
Q
A

The correct answer is:

a) Which agent develops a toxin that results in the specific modification of a cellular G-protein involved in signal transduction? → Bordetella pertussis,
b) Which is the most common aetiological agent in community-acquired pneumonia in elderly people with normal immune function? → Streptococcus pneumoniae,
c) Which organism has a cell wall containing mycolic acid, the latter being the basis for the acid-fast staining technique? → Mycobacterium tuberculosis

178
Q
A

The correct answer is: increase K+ channel opening in pacemaker cells and slow the rate of spontaneous depolarisation.

179
Q
A

The correct answer is: Increased capillary permeability

180
Q
A

The correct answer is: as more O2 molecules bind to the haemoglobin.

181
Q
A

The correct answer is: Renal artery

182
Q
A

The correct answers are: The cardiac silhouette occupies less than 50% of the chest diameter, Two thirds of the heart lies on the right side of the chest

183
Q
A

A balloon venous catheter is passed through the right heart and as far into the pulmonary arterial circulation as it will go. Balloon inflation wedges the catheter in a pulmonary small artery, isolates the tip from the pressure of the pulmonary artery behind it and creates a continuous link through the pulmonary capillaries and veins to the left atrium. It measures “pulmonary capillary wedge pressure”.

The correct answer is: Left atrial filling pressure

184
Q
A

Mid diastolic murmur suggests a defect that is evident during filling (diastole). The location of the sound (aligned with the apex of the heart) suggests that it is the mitral valve. Mitral valve incompetence (incorrect) would manifest as a defect during ejection. Symptoms suggest heart failure and this is likely caused by the valve defect.

The correct answer is: Mitral valve stenosis

185
Q
A

In a randomised control trial, one can be most certain that any factors influencing the outcome in individuals are not systematically distributed to one treatment group and that things that the subjects have done / taken that might have influenced their final outcome are similarly distributed. It provides the best insurance that the result was due to the intervention.

The correct answer is: Randomised -controlled trial (RCT)

186
Q
A

The correct answer is: is widely accepted by professional opinion in Australia.

187
Q
A

Reducing burden of disease refers to the greatest benefit in terms of years of life saved on a population basis. Smoking contributes the highest total burden of disease in Australia followed by high blood pressure and obesity, however the question relates specifically to IHD and stroke. The health burden of smoking has a major component as risk of lung cancer, and obesity has the added burden of diabetes,colorectal cancer and osteoarthritis.

The correct answer is: High blood pressure

188
Q
A

Anterior infarct identified by ST-segment elevation in leads V2, V3. Inferior (E) infarct would be identified in leads II, III, & aVF; lateral (D) infarct would be identified in leads V4-V6; while a septal infarct (C) would be identified in leads V1, V2 and might lead to additional conduction block (no evidence in ECG) – trial ischaemia (B) is unlikely to produce the symptoms but may lead to supraventricular arrhythmia, driving disturbance of ventricular rhythm (no evidence), and would not lead to the ST-segment elevation indicative of ventricular damage observed in the ECG trace.

The correct answer is: Region A

189
Q

A 73-year-old man says he has been increasingly suffering from shortness of breath, and you observe that he has swollen ankles. You suspect cardiac failure and order an X-ray because you wish to calculate his -cardiac transverse ratio (CT index) as an index of heart size.

Which labelled distances are needed to calculate his CT index?

A

To calculate the CT index, the maximum distance of the right border of the heart from the midline (line V) is added to the maximum distance of the left border of the heart from the midline (line Y), then this total is divided by the internal diameter of the chest at the level of the right hemidiaphragm (line Z).

The correct answer is: V, Y and Z

190
Q
A

This artery is at the “corner” of the axillary artery and you can see it’s branches clearly on the right side supplying structures superior (clavicle), lateral (deltoid and acromian) and medial (pectoral) to the axillary artery itself. Costocervical trunk is very medial, the second trunk after internal thoracic artery, superior thoracic branches off the horizontal party of the axillary artery more medial, lateral thoracic branches off on the more vertical part and has clear medial only branches, subscapular branches off the vertical part, more inferiorly, giving off circumflex scapular and thoracodorsal branches.

The correct answer is: Thoracoacromial artery

191
Q

A right ventricular infarction that affects the identified structure in the diagram may contribute to which of the following insufficiencies?

Select one:

Anterior and posterior mitral cusp insufficiency

Anterior and posterior tricuspid cusp insufficiency

Anterior and septal mitral cusp insufficiency

Anterior and septal tricuspid cusp insufficiency

Posterior and septal tricuspid cusp insufficiency

A

Anwer: Anterior and posterior tricuspid cusp insufficiency.

The septomarginal trabecula (moderator band) is pinned. The band contributes to the anterior papillary muscle of the right ventricle. The papillary muscle attaches to the anterior and posterior tricuspid cusps of the tricuspid valve.

The correct answer is: Anterior and posterior tricuspid cusp insufficiency

192
Q

A 28-year-old male presents with chest pain, shortness of breath, rapid heart rate, rapid breathing, cough, and signs of cyanosis. He has a 13 year history of smoking.

From his chest X-ray, what is the most likely diagnosis?

Select one:

Bronchiectasis

Emphysema

Pleurisy

Pneumonia

Pneumothorax

A

The correct answers are: Bronchiectasis, Pneumothorax

193
Q

The diagram above records the lung volumes and capacities from a normal individual during tidal breathing and maximum inspiration and forced expiration.

In a patient with chronic bronchitis, which labelled volume or capacity is most likely to be larger than the predicted normal value?

Select one:

A

B

C

D

E

A

In patients with airflow limitations, inadequate expiry time and increased airway resistance produce an increased end-expiry lung volume or functional residual capacity (FRC)

The correct answer is: B

194
Q

A new antihypertensive drug was tested by conducting an exercise stress test before and after it was administered.

On the basis of the heart rate (HR) and stroke volume (SV) responses to exercise shown in the graph, select from the list,.the pharmacological class of antihypertensive drug it most likely belongs to:

Select one:

Adrenergic alpha-receptor antagonist

Adrenergic beta-receptor antagonist

Angiotensin converting enzyme inhibitor

Calcium channel blocker

Loop diuretic

A

The correct answer is: Adrenergic beta-receptor antagonist

195
Q

A 15-year-old girl with a history of asthma undergoes a respiratory function assessment which demonstrates acute limitations in her FEV1.

Which segment of the respiratory system is most subject to dynamic airway collapse to limit the FEV1?

Select one:

Segment A

Segment B

Segment C

Segment D

Segment E

A

The correct answer is: Segment D

196
Q

The image of Heart Y below was taken at autopsy from a patient who died of heart failure; heart X is normal for comparison.

Heart Y is most likely representative of a heart subjected to …

Select one:

acute myocardial infarction.

chronic anaemia.

chronic aortic stenosis.

chronic mitral insufficiency.

pulmonary hypertension.

A

Heart Y represents mainly concentric hypertrophy (increased heart mass due to increased wall thickness with a normal chamber size). This response is typical of pressure overload such as occurs in aortic stenosis. Chronic anaemia and mitral valve insufficiency produce volume overload and more eccentric (dilated) hypertrophy. Myocardial infarction would also induce compensatory hypertrophy in the long term but also a section of thin (scarred myocardium would be evident.

The correct answer is: chronic aortic stenosis.

197
Q
A

A patent ductus arteriosus allows oxygenated left ventricular blood to return under pressure to the pulmonary artery (increase pulmonary artery pressure). The left ventricle must pump more blood per minute to maintain cardiac output and mean arterial pressure. Oxygen saturation of the pulmonary artery is increased and blood returning from the lungs remains fully saturated (systemic arterial O2 saturation).there is no effect on right ventricular filling although emptying would be impeded by the high pulmonary artery pressure

The correct answer is: increased left ventricular volume load.